Show that a_n --> 2: Using Calculus

  • Thread starter quasar987
  • Start date
  • Tags
    Tower
In summary, the homework statement states that if a_n is the sum of the squares of the integers from 1 to n, then a_n-->2. However, when looking at the sequence b_n=2-(a_n), it is clear that b_n-->0, meaning that a_n cannot be the sum of the squares of integers. Therefore, a_n must converge to 2 by induction.
  • #1
quasar987
Science Advisor
Homework Helper
Gold Member
4,807
32

Homework Statement


Let [itex]a_1=\sqrt{2}[/itex], [itex]a_2=(\sqrt{2})^{a_1}[/itex],... [itex]a_{n+1}=(\sqrt{2})^{a_n}[/itex]. Show that a_n-->2. you may use any relevant fact from calculus.

The Attempt at a Solution


I noticed that [itex]\ln(a_1)=\ln(\sqrt{2})=\ln(2)/2[/itex], [tex]\ln(a_2)=\ln((\sqrt{2})^{\ln(2)/2})=(\ln(2)/2)^2[/tex], ..., [itex]\ln(a_n)=(\ln(2)/2)^n[/itex]. But ln(2) < 2. So ln(2)/2 < 1, and thus ln(a_n)-->0 <==> a_n-->1. What's wrong?
 
Last edited:
Physics news on Phys.org
  • #2
quasar987 said:

Homework Statement


Let [itex]a_1=\sqrt{2}[/itex], [itex]a_2=(\sqrt{2})^{a_1}[/itex],... [itex]a_{n+1}=(\sqrt{2})^{a_n}[/itex]. Show that a_n-->2. you may use any relevant fact from calculus.

The Attempt at a Solution


I noticed that [itex]\ln(a_1)=\ln(\sqrt{2})=\ln(2)/2[/itex], [tex]\ln(a_2)=\ln((\sqrt{2})^{\ln(2)/2})=(\ln(2)/2)^2[/tex], ..., [itex]\ln(a_n)=(\ln(2)/2)^n[/itex]. But ln(2) < 2. So ln(2)/2 < 1, and thus ln(a_n)-->0 <==> a_n-->1. What's wrong?

[tex]\log a_2 = \log (\sqrt{2})^{a_1} [/tex]

not

[tex]\log a_2 = \log (\sqrt{2})^{\log a_1} [/tex]
 
  • #3
we know a_n= (sqrt(2))^{a_{n-1}}

eugh.

Suppose there is a limit. Let that limit be a. Can you show from this that a=2? I.e. what equality does a satisfy, and thus what is a (by inspection).

Now we sjust need to show that a_n converges to anything? What are the only things we know converge for sure? increasing bounded above sequences, or decreasing bounded below ones.
 
  • #4
Did someone say... proof by induction!?:biggrin:
 
  • #5
quasar987 said:

Homework Statement


Let [itex]a_1=\sqrt{2}[/itex], [itex]a_2=(\sqrt{2})^{a_1}[/itex],... [itex]a_{n+1}=(\sqrt{2})^{a_n}[/itex]. Show that a_n-->2. you may use any relevant fact from calculus.

One way to do this is to look at the sequence:
[tex]b_n=2-(a_n)[/tex]
So if [itex]b_n \rightarrow 0[/itex] you're good. Maybe you can look at:
[tex]\frac{b_n}{b_{n+1}}[/tex].
 
  • #6
matt grime said:
Suppose there is a limit. Let that limit be a. Can you show from this that a=2? I.e. what equality does a satisfy, and thus what is a (by inspection).
Nice. Suppoes a_n-->a. Then [itex]a_{n+1}=\sqrt{2}^{a_n}\rightarrow a[/itex]. But also, [itex]\sqrt{2}^{a_n}\rightarrow \sqrt{2}^a[/itex]. So [itex]\sqrt{2}^a=a[/itex]. So a=2.

NateTG said:
One way to do this is to look at the sequence:
[tex]b_n=2-(a_n)[/tex]
So if [itex]b_n \rightarrow 0[/itex] you're good. Maybe you can look at:
[tex]\frac{b_n}{b_{n+1}}[/tex].

Did you follow through with that idea? Cuz it did cross my mind, but I rejected it quickly because that [tex]\frac{b_n}{b_{n+1}}[/tex] fraction is not too cooperative as far as i can see.
 
Last edited:
  • #7
How could we show the sequence is bounded though? :/
 
  • #8
quasar987 said:
Did you follow through with that idea? Cuz it did cross my mind, but I rejected it quickly because that [tex]\frac{b_n}{b_{n+1}}[/tex] fraction is not too cooperative as far as i can see.

[tex]b_{n+1}=2-a_{n+1}=2-\sqrt{2}^{2-b_n}=2-\frac{2}{\sqrt{2}^{b_n}}[/tex]
I'm pretty sure it's possible to show that
[tex]0 < \frac{2-\frac{2}{\sqrt{2}^{b_n}}}{b_n} < 1[/tex]
if [itex]b_n \in (0,1)[/tex]
 
Last edited:
  • #9
quasar987 said:
How could we show the sequence is bounded though? :/

One method is guessing what the bound is, and then proving that the sequence never exceeds the bound.
 
  • #10
quasar987 said:
Nice. Suppoes a_n-->a. Then [itex]a_{n+1}=\sqrt{2}^{a_n}\rightarrow 2[/itex]. But also, [itex]\sqrt{2}^{a_n}\rightarrow \sqrt{2}^a[/itex]. So [itex]\sqrt{2}^a=a[/itex]. So a=2.
That doesn't follow at all. in fact you appear to be supposing the answer, unless it is just a typo, which seems most likely. If a_n converges to a, then a satisfies a=sqrt(2)^a.
 
  • #11
quasar987 said:
How could we show the sequence is bounded though? :/
I'm thinking induction-- which would just be what Hurkyl hinted at.
[tex]a_1 = \sqrt{2}^{\sqrt{2}} < 2[/tex] => true for n=1.
Assume true for n, i.e. [tex]a_n = \sqrt{2}^{a_n -1} < 2[/tex]... and show that this implies n+1 true, which would be something like:
[tex]a_{n+1} = \sqrt{2}^{a_n} < \sqrt{2}^{2} = 2[/tex]
Or something.
And it's a monotone increasing sequence, so...
 
Last edited:
  • #12
Yeah I realized induction does the trick at work today :smile:(typo corrected in post #6)
 
  • #13
matt grime said:
That doesn't follow at all. in fact you appear to be supposing the answer, unless it is just a typo, which seems most likely. If a_n converges to a, then a satisfies a=sqrt(2)^a.


That's the point though... IF an is a convergent sequence, by algebra of limits, it must converge to 2 by the argument given. This is always a good way to start, because often to show a sequence converges, it's best to know what it converges to. For example, in this case if we didn't have the number 2, it would be difficult to pull the number out of mid-air and show it's an upper bound of the sequence (which it is regardless of whether we've verified it's going to be the limit, so there's no circular logic); so starting by showing it converges to something and then finding the limit would have been a much harder way to progress
 
  • #14
What guarantees us once we're at

[tex]a^{1/a}=2^{1/2}[/tex]

that a=2 is the only solution?
 
  • #15
quasar987 said:
What guarantees us once we're at

[tex]a^{1/a}=2^{1/2}[/tex]

that a=2 is the only solution?
It's not. a=4 works too.

I guess you're going to need to use some bounds.
 
  • #16
How do we methodically find all the solutions to this?
 
  • #17
quasar987 said:
How do we methodically find all the solutions to this?

[tex]\frac{d}{da}a^{\frac{1}{a}}=\frac{1-\ln a}{a^2}a^{\frac{1}{a}}[/tex]
Since the derivative only changes sign once for [itex]a>0[/itex], those are the only positive real solutions. I expect that there is an infinite number of complex solutions.
 

Related to Show that a_n --> 2: Using Calculus

What does it mean for a sequence to converge to a limit of 2?

Convergence of a sequence means that as the terms of the sequence continue, they get closer and closer to a certain value, in this case 2. This does not necessarily mean that the last term of the sequence is exactly 2, but that it gets closer and closer to 2 as the sequence progresses.

How can I use calculus to show that the limit of a sequence is 2?

Calculus involves using mathematical methods to analyze and understand changing quantities. To show that a sequence has a limit of 2, you can use the concept of limits and apply it to the sequence. This involves using the formal definition of a limit and solving for the limit value, in this case 2.

What are the necessary conditions for a sequence to converge to a limit of 2?

In order for a sequence to converge to a limit of 2, it must satisfy two conditions: the sequence must be bounded, meaning that the terms do not get infinitely large or small, and the sequence must be monotonic, meaning that the terms either consistently increase or decrease as the sequence progresses.

Can calculus be used to show convergence of any sequence?

Yes, calculus can be used to show the convergence of any sequence as long as the sequence satisfies the necessary conditions for convergence. However, some sequences may be more complex and require more advanced calculus techniques to prove convergence.

Are there any other methods besides calculus to show that a sequence converges to a limit of 2?

Yes, there are other methods such as using the definition of convergence, the squeeze theorem, or the Cauchy criterion. However, calculus is often the most efficient and widely used method to prove convergence of a sequence.

Similar threads

  • Calculus and Beyond Homework Help
Replies
14
Views
446
  • Calculus and Beyond Homework Help
Replies
5
Views
361
  • Calculus and Beyond Homework Help
Replies
9
Views
778
  • Calculus and Beyond Homework Help
Replies
1
Views
423
Replies
5
Views
1K
  • Calculus and Beyond Homework Help
Replies
12
Views
1K
  • Calculus and Beyond Homework Help
Replies
11
Views
1K
  • Calculus and Beyond Homework Help
Replies
12
Views
1K
  • Calculus and Beyond Homework Help
Replies
5
Views
1K
  • Calculus and Beyond Homework Help
Replies
8
Views
1K
Back
Top